13

I'm trying to prove or disprove the following conjecture:

If $f(x)$ is continuous in $(0,1]$ and $f(x)\to\infty$ as $x\to 0^+$ then $L=\lim\limits_{n\to\infty}\sum\limits_{k=1}^n f\left(\frac{k}{n}\right)$ does not exist.

My attempt

I tried proof by contradiction. Asssume $L$ exists.

$L=\lim\limits_{n\to\infty}n(\frac{1}{n})\sum\limits_{k=1}^n f\left(\frac{k} {n}\right)=\left(\lim\limits_{n\to\infty}n\right)\int_{0}^1 f(x)dx$

(EDIT: As mentioned by @FShrike in the comments, the previous step is not valid.)

$\therefore \int_{0}^1 f(x)dx=0$

There are functions $f(x)$, continuous in $(0,1]$, such that $f(x)\to\infty$ as $x\to 0^+$ and $\int_{0}^1 f(x)dx=0$ . For example, $f(x)=-\ln{x}-1$, in which case $L$ does not exist, by Stirling's approximation.

But I do not see why all such functions $f(x)$ would imply that $L$ does not exist.

Context:

I am interested in geometrical infinite products (example1, example2). The conjecture in this question, via the substitution $f(x)=-\ln{g(x)}$, is equivalent to: If $g(x)$ is continuous in $(0,1]$ and $\lim\limits_{x\to 0^+}g(x)=0$ then $\lim\limits_{n\to\infty}\prod\limits_{k=1}^ng\left(\frac{k}{n}\right)$ either equals $0$ or does not exist, which stands in interesting contrast with the fact that infinite products of lengths or areas, that tend to $0$, can equal a positive number.

EDIT2:

I'm not sure if this is helpful, but I have noticed that $L_2=\lim\limits_{n\to\infty}\sum\limits_{k=1}^n f\left(\frac{k-1/2}{n}\right)$ can exist.

For example, $\lim\limits_{n\to\infty}\sum\limits_{k=1}^n \left(-\ln{\left(\frac{k-1/2}{n}\right)}-1\right)=-\frac{\ln{2}}{2}$. (Another question of mine yielded methods for dealing with the sum $\sum\limits_{k=1}^n \ln{(k-\frac12)}$.)

I do not understand why replacing $k$ with $k-\frac12$ seems to make the limit existable (if that's a word).

Dan
  • 22,158
  • 1
    Are you assuming continuity or Riemann integrability? – dfnu Oct 28 '22 at 12:44
  • @dfnu I don't think there is a counter-example, whether continuous or otherwise. But I'll go ahead and add the assumption of continuity, since I'm interested in continuous curves. – Dan Oct 28 '22 at 12:58
  • If I understand correctly the assumptions, it can definitely hold that the limit exists and it is infinite. Take $f(x) = \frac1x$. Then $\sum_{k=1}^n f\left(\frac{k}n\right)=\sum_{k=1}^n \frac{n}k > n$ – dfnu Oct 28 '22 at 12:59
  • 1
    @dfnu I regard a (so-called) infinite limit as "does not exist". – Dan Oct 28 '22 at 13:03
  • 2
    You can't assume the partial sums tend to an integral; the integral itself may not exist. Factoring out $(\lim_n n)$ is invalid anywya – FShrike Oct 28 '22 at 13:39
  • @FShrike Good points. – Dan Oct 28 '22 at 13:57
  • 3
    If you remove continuity from your hypotheses, there are in fact counter examples. Take $$f(x) = \begin{cases} \cot(\pi x) & (0<x<1)\ 0 & (x=0 \lor x=1) \end{cases}$$ – dfnu Oct 28 '22 at 14:23
  • 1
    @Zerox I guess we are talking here about continuity in $(0,1]$ since $f(x)$ diverges in $0$. The integral therefore may not exist. My example gives a function which is not continuous there. – dfnu Oct 28 '22 at 15:03
  • 1
    For your example function, I think using Stirling's formula, $\sum_{k=1}^n f(k/n)$ would be asymptotic to $-\frac{1}{2} \log (2\pi n)$ so in particular it approaches $-\infty$. (I could very well have made a computational error.) – Daniel Schepler Oct 28 '22 at 16:38
  • If $f\geq0$ over $(0,1]$, the statement is trivial since $\sum^n_{k=1}f(k/n)\geq f(1/n)\xrightarrow{n\rightarrow\infty}\infty$. For the general case, for $A>0$ let $0<a<1$ such that $f(x)>A$ for all $0<x\leq a$. Then $\sum^n_{k=1}f(k/n)\sim \sum^{\lfloor na\rfloor}_{k=1}f(k/a)-n\int^1_af$. If $\int^1_a|f|<\infty$ then I am sure it its possible to show that the statement still holds. – Mittens Oct 28 '22 at 18:16
  • @OliverDíaz I think the main point is what happens of the positive part. Consider as two "opposite" examples $f(x) = -\log x -1$ and $f(x) = \frac1x -1$. In the first case the $n$-th sum is $\log\frac{n^n}{n!}-n$ which tends to $-\infty$, in the second case it is $n H_n -n$ which tends to $+\infty$. – dfnu Oct 28 '22 at 18:48
  • Now posted to MO, https://mathoverflow.net/questions/434707/does-there-exist-a-continuous-function-fx-such-that-f0-0-and-0-lim-n – Gerry Myerson Nov 16 '22 at 21:52

7 Answers7

2

The statement in the title of this question can be disproved. I posted an equivalent question on Math Overflow, and it has been answered.

Note: The $f(x)$ in this question, and the $f(x)$ in the Overflow question, are different. Here is how they are related:

$$[f(x)\text{ in this question}]=-\ln{[f(x)\text{ in the Math Overflow question}]}$$

Dan
  • 22,158
1

This is more of a comment but it highlights the fallacy of claiming that OP implies $L =\infty$ which is simply not true. Consider $f(x)=x^{-1/2}, 0 < x \le 1/16$ and then $f(x)$ linear going from $4$ to $-100$ on $[1/16, 1/8]$ (so $-1664x+108$) and $f(x)=-100, 1/8 \le x \le 1$.

Let $n=16m$ then $\sum_{k=1}^{16m}f(k/n)=\sum_{k=1}^{m}\sqrt {16m/k}+\sum_{k=m+1}^{2m}(-104k/m+108)+\sum_{k=2m+1}^{8m}(-100)$

But $\sum_{k=1}^{m}1/\sqrt k \le 1+\int_1^{m}dx/\sqrt x=2\sqrt m -1$ so $\sum_{k=1}^{m}\sqrt {16m/k} \le 8m$

$\sum_{k=m+1}^{2m}(-104k/m+108)=-104(2m+1)+104(m+1)/2+108m=-48m-52$

$\sum_{k=2m+1}^{8m}(-100)=-600m$ so it is obvious that the original sum is highly negative and indeed goes to $-\infty$

More generally it is not hard to show that if $f$ monotonic on $[0,1]$ (the above example is so) and $\int_0^1f(x)dx$ finite (integral being the Lebesgue one or if you want $\lim_{\epsilon \to 0}\int_{\epsilon}^1f(x)dx$ exists in our case with the latter integral being Riemann) then $$\frac{\sum_{k=1}^nf(k/n)}{n} \to \int_0^1f(x)dx$$ which immediately shows that if the integral is negative and the function decreasing, the sums in the OP go to infinity

I see nothing to suggest that we may not get examples where things balance nicely and the limit is finite or at least the sums are bounded.

Conrad
  • 27,433
1

This is more of a long comment than a solution. Since $f$ is continuous and $\lim_{x\rightarrow0+}f(x)=\infty$, $I=\int^1_0f=\infty$ or $I=\int^1_0f$ is finite. I propose to split the interval $(0,1]$ in two pieces so that on $(0,a]$, $f>1$, and $\int^1_af$ has the same sign as $I$ (or $<0$ if $I=0$).

The sum $\sum^n_{k=\lfloor na\rfloor +1}f(k/n)\sim n\int^1_af$ in the sense that $b_n:=\frac{\sum^n_{k=\lfloor na\rfloor +1}f(k/n)}{n\int^1_a}\xrightarrow{n\rightarrow\infty}1$. We then consider $$R_n:=\frac{\sum^n_{k=1}f(k/n)}{n\int^1_af}=\frac{\tfrac1n\sum^{\lfloor na\rfloor}_{k=1}f(k/n)}{\int^a_1f}+b_n$$

All members in the sum $\sum^{\lfloor na\rfloor}_{k=1}f(k/n)$ are positive.

If $f$ is monotone nonincreasing in $(0,a]$ and integrable, then $$R_n\xrightarrow{n\rightarrow\infty}\frac{\int^a_0f}{\int^1_af}+1=\frac{\int^1_0f}{\int^1_af}=R$$ If $I\neq0$, then $$S_n:=\sum^n_{k=1}f(k/m)\xrightarrow{n\rightarrow\infty}\operatorname{sign}(I)\cdot\infty$$

If $f$ is monotone nonincreasing in $(0,a]$ and $I=\infty$, then $$R_n\xrightarrow{n\rightarrow}\operatorname{sign}\Big(\int^1_af\big)\cdot\infty$$ and so, $S_n\xrightarrow{n\rightarrow\infty}\infty$

This suggests that the case $\int^1_0f=0$ is where the interesting stuff happens.

I also think that understanding the case where $f$ is monotone non increasing will yield more information even in the general case for we can consider the monotone envelops \begin{align} \alpha(x)=\inf\{f(t): 0<t\leq x\}\qquad \beta(x)=\sup\{f(t): x\leq t\leq 1\} \end{align} $\alpha$ and $\beta$ are both monotone non increasing and $$\alpha\leq f\leq \beta$$

Mittens
  • 39,145
  • The problem is that we can easily make $\frac{\tfrac1n\sum^{\lfloor na\rfloor}_{k=1}f(k/n)}{\int^a_1f}$ to go to $1$ with simple examples but the difference still may go to $\pm \infty$ so things really have to balance finely – Conrad Oct 28 '22 at 21:16
  • The problem is that $\sum^n_{k=\lfloor na\rfloor +1}f(k/n)\sim n\int^1_af$ is not really an additive asymptotic, only a multiplicative one so $\sum^n_{k=\lfloor na\rfloor +1}f(k/n)-n\int^1_af$ can be very big in absolute value (as big as any $o(n)$) – Conrad Oct 28 '22 at 21:34
  • also not so sure about the claim about $I=0$ being the critical case for nonmonotonic functions (think if you take $f(x)=x^{-1/2}(|\cos 2\pi/x|+1)$ near $0$ where the integral is bounded by a constant but the Riemann sums can oscillate so you may get some that are unnaturally big because of big special values at rationals so they may need a much bigger negative second part to compensate than what you get from the total integral) – Conrad Oct 28 '22 at 21:43
  • modified it so you go to infinity- the point remains though; on the other hand for the OP, that is really not that much of interest imho in the sense that if you could prove that the limit must be $\pm \infty$ for a monotonic $f$ with the required properties and i would be happy; I still believe that if not finite limit, at least bounded sums are possible – Conrad Oct 28 '22 at 21:46
  • @Conrad: As for you second comment, $b_n:=\sum^n_{k=[na]+1}f(k/n)/\Big(n\int\int^1_af \Big)\xrightarrow{n\rightarrow\infty}1$. In my posing I used $\sim$ in my last equation. Than can be replaced by $=$ and $1$ by $b_n$. – Mittens Oct 28 '22 at 21:50
  • I agree with that but unfortunately in the OP we have differences (in other words we need to deal with $\sum f(k/n)$ so if we replace the latter part by an integral we have an error $\sum_{k >[na]}f(k/n)-n\int_a^1f$ and as noted this error can be as big as any $o(n)$ – Conrad Oct 28 '22 at 21:52
  • @Conrad: sorry for the problems with LaTeX. What I am suggesting is to estimate the limit $\frac{\sum^{\lfloor na\rfloor}_{k=1}f(k/n)}{n\int^1_af}+b_n$. If it different from $0$, then the original sum converges to either $\infty$ to $-\infty$; the case in which this limit is zero is, I believe, the challenging one. – Mittens Oct 28 '22 at 22:10
1

First consider $g(x)$ such that $g(x)=1$ on $(2^{-3},2^{-1}]$, $g(x)=2$ on $(2^{-5}, 2^{-3}]$, $g(x) = 4$ on $(2^{-7}, 2^{-5}]$, etc. Taking them from rightmost on the number line first and going to the left, these intervals fill up the space from $0.5$ to $0$ and each one is one-fourth the size of the previous one. So in the limit, each one will have one-fourth the points as the one before, and twice the value, for double the total value. Thus, the total value over all of them will tend towards $2$ raised to the number of intervals. But the number of intervals with at least one value will tend towards $\log_2(n)$, to the total value will tend towards a linear function of $n$. If we give the interval from $0.5$ to $1$ a constant value, then the total of values in that interval will also tend towards a linear function of $n$. So we just have those two functions cancel out to some finite value.

Now, this does have the issue that $g(x)$ is not continuous, but I think that can be dealt with. We can define $f$ to be “close” to $g$, and have it go from the value of one interval to the next in exponentially decreasing time.

Acccumulation
  • 12,210
  • I don't think your $f(x)$ satisfies the condition that $f(x)\to\infty$ as $x\to 0^+$, because we can find $x$ values arbitrarily close to $0$ such that $f(x)=0$. – Dan Oct 29 '22 at 00:19
  • Since $f$ nonnegative if it were going to get infinity at zero than $f(1/n) \to \infty$ already so the limit is infinite – Conrad Oct 29 '22 at 00:57
  • @Dan Yes, I got mixed up between unbounded and going to infinity. I've written up another attempt. – Acccumulation Oct 29 '22 at 19:38
  • It's easy to do a handwaving approach but not that easy to do a rigurous example since you need to estimate the additive errors to be $o(1)$ or maybe $O(1)$ if you want just bounded sums and that is non trivial; my example with square root inverse easily can be changed to cancel out the linear main term but the error is still going to infinity so one needs to be very careful - as noted I do believe a counterxample is achievable but things need to balance finely – Conrad Oct 29 '22 at 19:46
1

I would like to further elaborate on TravorLZH's answer.

Under the assumptions in that answer (i.e., $f(x)$ monotonic and continuously differentiable) we have that

$$\sum_{k=1}^nf\left(\frac kn\right)=n\int_{1/n}^1f(t)\mathrm dt+\frac12f\left(\frac1n\right)+\frac12f(1)+O\left(-{f'(1/n)\over n}\right)+O\left(\frac1n\right).$$

Thus if we further assume that $$\lim_{x\to 0^+} x f'(x) = L_1 < \infty$$ The LHS converges iff $$\lim_{x\to 0^+}\frac1x \int_x^1 f(t) dt +\frac12 f(x) = L_2 <\infty.$$

If $F(x)$ is a primitive of $f(x)$ we have $$g(x) = \frac1x \left[F(1)-F(x)\right] + \frac12 F'(x)$$ with $\lim_{x\to 0^+} g(x) = L_2$. The solutions to this differential equation are \begin{eqnarray} F(x) &=& -x^2 \int_1^x \left(-\frac{2F(1)}{t^3}+\frac{g(t)}t\right)dt + C_1x^2=\\ &=&F(1) + x^2\int_1^x \frac{g(t)}{t^2}dt + C_2x^2. \end{eqnarray} Differentiating yields $$f(x) = 2x\int_1^x \frac{g(t)}{t^2} dt + g(x) +2C_2x.$$ Since $\lim_{x\to 0^+}g(x)=L_2$, there exists $$G = \sup_{x\in(0,1]}g(x).$$ Also $$\lim_{x\to 0^+} f(x) = -\lim_{x\to 0^+}x^2 \int_x^1 \frac{g(t)}{t^2}dt+L_2.$$

Fix $\varepsilon > 0$ and select $\delta > 0$ in such a way that $|g(x)| < |L_2| + \varepsilon$, for $0< x<\delta$, then we have \begin{eqnarray} |f(x)| &<& x^2 \int_x^\delta \frac{|L_2|+\varepsilon}{t^2} dt+ \delta^2 (1-\delta)\frac{G}{\delta^2} + |L_2|\leq\\ &<&2 x^2\frac{|L_2|+\varepsilon}{x} + (1-\delta)G + |L_2|< \\ &<& 2\delta(|L_2|+\varepsilon) + (1-\delta)G + |L_2| \end{eqnarray} But this contradicts the hypothesis that $\lim_{x\to 0^+} f(x) = +\infty$.

In conclusion, if $f(x)$ is monotonic and continuously differentiable, the sum in OP cannot converge if $$\lim_{x\to 0^+} xf'(x) =L_1 < \infty$$

dfnu
  • 7,528
1

This answer discusses the situation when we restrict the consideration to the situation when $f$ is decreasing, $f'$ is monotonic and continuously differentiable.

Applying Euler-Maclaurin formula, we get

$$ \sum_{k=1}^nf\left(\frac kn\right)=n\int_{1/n}^1f(t)\mathrm dt+\frac12f\left(\frac1n\right)+\frac12f(1)+\frac1n\int_1^n\overline B_1(t)f'\left(\frac tn\right)\mathrm dt, $$

where $\overline B_1(t)=t-\lfloor t\rfloor-\frac12$ is the first Bernoulli function.

Integration by parts on the rightmost integral gives

$$ \int_1^n\rho(t)f'\left(\frac tn\right)\mathrm dt=\left.\frac12\overline B_2(t)f'\left(\frac tn\right)\right|_1^n-{1\over2n}\int_1^n\overline B_2(t)f''\left(\frac tn\right)\mathrm dt=O\left\{-f'\left(\frac1n\right)\right\}+O(1) $$

Combining everything, we have

$$ \sum_{k=1}^nf\left(\frac kn\right)=n\int_{1/n}^1f(t)\mathrm dt+\frac12f\left(\frac1n\right)+\frac12f(1)+O\left(-{f'(1/n)\over n}\right)+O\left(\frac1n\right).\tag1 $$

Now, let $f(x)=-\ln x-1$, so that it satisfies the conditions for (1). Calculation gives

$$ \int_{1/n}^1f(t)\mathrm dt=-x\ln x|_{1/n}^1=\frac1n\ln\frac1n,\quad -f'\left(\frac1n\right)=\frac1n, $$

so (1) becomes

$$ \sum_{k=1}^nf\left(\frac kn\right)=\ln\frac1n+\frac12\left(-\ln\frac1n-1\right)+O\left(n\over n\right)+O\left(\frac1n\right)=\frac12\ln\frac1n+O(1), $$

which indicates that the limit does not necessarily exist even when $\int_0^1f(t)\mathrm dt=0$.

TravorLZH
  • 6,718
  • In your last line, do you mean "the limit does not necessarily exist", OR "the limit necessarily does not exist"? – Dan Oct 30 '22 at 00:18
  • I'm saying that the limit "does not necessarily exist" because the choice $f(x)=-\ln x-1$ is one example where that limit does not exist. – TravorLZH Oct 30 '22 at 00:25
  • Is then correct to say that we need, for the sum to converge, $$xf'(x) \to L_1 < \infty$$ and $$\frac1x \int_x^1 f(t)dt+\frac12 f(x) \to L_2 < \infty$$ for $x\to 0$?

    With $f(x) = -\log x -1$ the second condition is not satisfied, then.

    – dfnu Oct 31 '22 at 12:11
  • I would believe that as the $\frac12f(x)$ term actually comes from trapezoid estimates – TravorLZH Nov 01 '22 at 17:22
1

If we assume that $f(x)$ is decreasing, here is an argument that $L=\lim\limits_{n\to\infty}\sum\limits_{k=1}^n f\left(\frac{k}{n}\right)$ does not exist.

enter image description here

The graph shows an example with $n=3$. (The curve does not have to be above the x-axis.)

$\begin{align} \frac{1}{2n}\sum\limits_{k=1}^{2n}f\left(\frac{k}{2n}\right) & =\frac{1}{n}\sum\limits_{k=1}^{n}f\left(\frac{k}{n}\right)+\color{red}{(\text{area of red region})}+\color{blue}{(\text{total area of blue regions})}\\ & =\frac{1}{n}\sum\limits_{k=1}^{n}f\left(\frac{k}{n}\right)+\color{red}{\frac{1}{2n}\left(f\left(\frac{1}{2n}\right)-f\left(\frac{1}{n}\right)\right)}+\color{blue}{\frac{1}{2n}\left(\frac{1}{2}\left(f\left(\frac{1}{n}\right)-f(1)\right)\right)+O\left(\frac{1}{n}\right)} \end{align}$

Multiply by $2n$ and rearrange.

$2\sum\limits_{k=1}^{n}f\left(\frac{k}{n}\right) - \sum\limits_{k=1}^{2n}f\left(\frac{k}{2n}\right) =\color{brown}{\frac{1}{2}f\left(\frac{1}{n}\right) - f\left(\frac{1}{2n}\right)}+\frac{1}{2}f(1)+c+O\left(\frac{1}{n^2}\right)$

$f(x)\to\infty$ as $x\to0^+ \implies \color{brown}{\frac{1}{2}f\left(\frac{1}{n}\right) - f\left(\frac{1}{2n}\right)} \le -\frac{1}{2}f\left(\frac{1}{2n}\right)$

$\therefore 2\sum\limits_{k=1}^{n}f\left(\frac{k}{n}\right) - \sum\limits_{k=1}^{2n}f\left(\frac{k}{2n}\right) \le -\frac{1}{2}f\left(\frac{1}{2n}\right)+\frac{1}{2}f(1)+c+O\left(\frac{1}{n^2}\right)$

If $L=\lim\limits_{n\to\infty}\sum\limits_{k=1}^n f\left(\frac{k}{n}\right)$ exists, then as $n\to\infty$, LHS $\to L$ but RHS $\to-\infty$. Contradiction.

$\therefore L$ does not exist.

Dan
  • 22,158
  • In your graphical example $\int^1_0f>0$ and $f$ is monotone decreasing in which case things are easy to check (limit os sum diverges to $\infty$. The difficulty arises when either $\int^1_0f=0$ or when $f$ oscillates a lot near $0$. – Mittens Oct 30 '22 at 03:21
  • 1
    @OliverDíaz, I agree that my argument requires that $f$ is decreasing (I'll edit that in). But does my argument really require that $\int^1_0f=0$? If $f$ dips below the x-axis, I think all the steps in my argument are still valid. – Dan Oct 30 '22 at 03:37
  • If $\int^1_0f\neq0$ and $f$ is monotone decreasing with $f(0+)=\infty$ the argument in my long comment (in the answer section) show that indeed, the sum $\sum^n_{k=1}f(k/n)$ diverges to wither $\infty$ or $-\infty$. The case $\int^1_0f$ being inconclusive. For more general $f$, even in the case $\int^1_0f\neq0$ it could be that the limit actually exists. – Mittens Oct 30 '22 at 03:53
  • 1
    @OliverDíaz Now there's a counterexample I'd like to see! – Dan Oct 30 '22 at 05:37